Difference between revisions of "2019 AMC 10A Problems/Problem 1"

(Problem 1)
(Solution)
Line 5: Line 5:
 
== Solution ==  
 
== Solution ==  
  
<math>2^{\left(0^{\left(1^9\right)}\right)}+\left(\left(2^0\right)^1\right)^9
+
<math>2^{\left(0^{\left(1^9\right)}\right)}+\left(\left(2^0\right)^1\right)^9</math>
  
=  1+1 = \boxed{2}</math> which corresponds to <math>\boxed{\text{C}}</math>.
+
<math>=  1+1 = \boxed{2}</math> which corresponds to <math>\boxed{\text{C}}</math>.
  
 
== See Also ==
 
== See Also ==

Revision as of 18:18, 19 January 2020

Problem 1

What is the value of \[2^{\left(0^{\left(1^9\right)}\right)}+\left(\left(2^0\right)^1\right)^9?\] $\textbf{(A) } 0 \qquad\textbf{(B) } 1 \qquad\textbf{(C) } 2 \qquad\textbf{(D) } 3 \qquad\textbf{(E) } 4$

Solution

$2^{\left(0^{\left(1^9\right)}\right)}+\left(\left(2^0\right)^1\right)^9$

$=  1+1 = \boxed{2}$ which corresponds to $\boxed{\text{C}}$.

See Also

2019 AMC 10A (ProblemsAnswer KeyResources)
Preceded by
First Problem
Followed by
Problem 2
1 2 3 4 5 6 7 8 9 10 11 12 13 14 15 16 17 18 19 20 21 22 23 24 25
All AMC 10 Problems and Solutions

The problems on this page are copyrighted by the Mathematical Association of America's American Mathematics Competitions. AMC logo.png